Math, asked by chintalapudisahasra, 1 day ago

express 1.729 bar in p q form​

Answers

Answered by MaheswariS
0

\underline{\textbf{Given:}}

\mathsf{1.\overline{729}}

\underline{\textbf{To find:}}

\mathsf{\dfrac{p}{q}\;form\;of\;1.\overline{729}}

\underline{\textbf{Solution:}}

\mathsf{1.\overline{729}}\;\textsf{is a non-terminating recurring decimal number}

\textsf{We can find its}\;\mathsf{\dfrac{p}{q}\;form\;as\;follows:}

\mathsf{Let\;x=1.729\,729\,729\;.\;.\;.\;.\;.}--------(1)

\textsf{Multiply bothsies by 1000, we get}

\mathsf{1000\,x=1729.729\,729\,729\;.\;.\;.\;.\;.}--------(2)

\textsf{Subtract (1) from (2), we get}

\mathsf{1000\,x-x=1728}

\mathsf{999\,x=1728}

\implies\mathsf{x=\dfrac{1728}{999}}

\implies\boxed{\mathsf{1.\overline{729}=\dfrac{1728}{999}}}

#SPJ3

Similar questions